PT1.S3.Q12 - Some of the most prosperous nations...

nbt19anbt19a Core Member
edited August 2021 in Logical Reasoning 6 karma

For a question like this that states "Which of the following indicates an error in the reasoning leading to the prediction", am I looking to attack a major premise instead of the conclusion? I got this question wrong and I am having difficulty discerning the difference between answer choice A with the correct answer D.

Comments

  • MissionLsatMissionLsat Member
    379 karma

    Here. the conclusion in the argument is " The nations, national saving will decrease if the average age of population continues to rise. And the support give for this is "Older people have fewer reasons to save than younger people."

    But this does not means that older people will be saving less or the younger will be saving more. Right. Older people would be having fewer reasons to save but that reason could be big enough to save a hello lot of money for lets say retirement plan.

    So in most of the questions it is the assumption, the missing gap that you would have to figure out in order to get to the right answer. So always try to predict.

    Answer choice A- The reasons that younger people have for saving money are irrelevant to are argument. So the stimulus lacking those reasons wouldn't be the flaw. Right. As it is simply irrelevant.

    Answer choice D- This is exactly what the error is in the reasoning. Even if the older people have fewer reasons to save, their reasons could be much more important which could require a large amount of savings. Thus requiring them to save more. Could still save more than the young ones.

  • Glutton for the LSATGlutton for the LSAT Alum Member
    edited August 2021 551 karma

    Hi @nbt19a

    Typically, in flaw questions, the flaw is in the reasoning (i.e., the premises) of the argument and how it relates back to the conclusion. In this case, one premise is that older people have fewer reasons to save relative to younger people. Another premise is a conditional: If the average age of prosperous nations rise, then it will reflect the trend of the first premise. This leads to the conclusion that the trend of prosperous nations experiencing a drop in savings will continue.

    Answer choice (A) does not effectively point out the mistaken relationship between the premise and conclusion. The premises in the stimulus deal with the quantitative analysis of reasons by age group (i.e., the amount of reasons different age groups have). Answer choice (A) is pointing to a qualitative analysis of reasons which is not mentioned in the stimulus, and, is in fact not relevant to the stimulus. Even if we were to identify the strongest reason that younger people have for saving money, how does that interact with the quantitative content of both premises? It doesn't.

    I hope this helps!

Sign In or Register to comment.